4. In preparation for the storm, Mike has printed out some flyers
advertising that he will shovel snow for $15 per hour. Mike also will
charge a minimum of $10.
a. Write an equation to find the total amount charged, y, after x
numbers of hours shoveling snow.

Answers

Answer 1

Answer:

y = 10 between 0 and 1

AND

y=15x for x>=1

Step-by-step explanation:

This has 2 segments. One when 0 < x < 1 (y=$10), and one when x >= 1 (y=$15x)

So

y = 10 between 0 and 1

AND

y=15x for x>=1


Related Questions

Find the 13th term of the arithmetic sequence -3x – 1,42 + 4,112 + 9, ...

Answers

Answer:

The 13th term is 81x + 59.

Step-by-step explanation:

We are given the arithmetic sequence:

[tex]\displaystle -3x -1, \, 4x +4, \, 11x + 9 \dots[/tex]

And we want to find the 13th term.

Recall that for an arithmetic sequence, each subsequent term only differ by a common difference d. In other words:

[tex]\displaystyle \underbrace{-3x - 1}_{x_1} + d = \underbrace{4x + 4} _ {x_2}[/tex]

Find the common difference by subtracting the first term from the second:

[tex]d = (4x+4) - (-3x - 1)[/tex]

Distribute:

[tex]d = (4x + 4) + (3x + 1)[/tex]

Combine like terms. Hence:

[tex]d = 7x + 5[/tex]

The common difference is (7x + 5).

To find the 13th term, we can write a direct formula. The direct formula for an arithmetic sequence has the form:

[tex]\displaystyle x_n = a + d(n-1)[/tex]

Where a is the initial term and d is the common difference.

The initial term is (-3x - 1) and the common difference is (7x + 5). Hence:

[tex]\displaystyle x_n = (-3x - 1) + (7x+5)(n-1)[/tex]

To find the 13th term, let n = 13. Hence:

[tex]\displaystyle x_{13} = (-3x - 1) + (7x + 5)((13)-1)[/tex]

Simplify:

[tex]\displaystyle \begin{aligned}x_{13} &= (-3x-1) + (7x+5)(12) \\ &= (-3x - 1) +(84x + 60) \\ &= 81x + 59 \end{aligned}[/tex]

The 13th term is 81x + 59.


What value of x is in the solution set of -2(3x + 2) >-8x + 6

Answers

Answer:

x >5

Step-by-step explanation:

-2(3x + 2) >-8x + 6

Distribute

-6x-4 > -8x+6

Add 8x to each side

-6x-4+8x > -8x+8x+6

2x-4 > 6

Add 4 to each side

2x-4+4 > 6+4

2x> 10

Divide by 2

2x/2 >10/2

x >5

Step-by-step:

-2(3x+2)>-8x+6. divided by 2,-3x-2>-4x+3. the solution set of -2(3x+3)>-8x+6 is {x/x>5}

Answer:

6

The function (1) describes the height, in feet, of an object at time, in seconds, when it is launched upward from the ground at an initial speed of 112 feet per second.

a. Find the domain.
b. What does the domain mean in this context?

Answers

Answer:

see below

Step-by-step explanation:

The domain is the values that the input takes

The values go from 0 to 7

0≤x≤7

This is the time from the initial launch until the object hits the ground

What is the solution for z?
24=z/0.6?

Answers

Answer:

[tex]z=14.4[/tex]

Step-by-step explanation:

One is given the following equation:

[tex]24=\frac{z}{0.6}[/tex]

Use inverse operations to solve this equation. Multiply both sides of the equation by (0.6) to undo the division of (0.6).

[tex]24=\frac{z}{0.6}[/tex]

[tex](0.6)(24)=z[/tex]

Simplify,

[tex](0.6)(24)=z[/tex]

[tex]z=14.4[/tex]

Monica took a survey of her classmates' hair and eye color. The results are in the table below.

Answers

0.14 would be correct

x3 + (y +z) factorize​

Answers

I think you wrote the question wrong this question is not factorable

Find the y-intercept of the following equation. Simplify your answer.
y = -10x -3/7

Answers

Answer: (0, -3/7)

Step-by-step explanation:

The Y-intercept would be the value of y when x is at 0, which is when the Y axis is intercepted. -3/7 is the starting position of the function when the the X = 0.

Rewrite
4/10 : 1/25 as a unit rate.

A: 10:1
B: 25:4
C: 2:125
D: 100:1

Answers

Answer:

4/10 : 1/25

4/10 / 1/25 = 4/10 x 25/1 = 100/10 = 10.

10 can also be written as 10:1, so A is correct.

Hope this helps!

The velocity of a particle moving along a straight line is given by v(t)=6t2+4t−5 cm/sec at time t seconds with initial position s(0)=3 cm. What is the position of the particle at t=2 seconds, in cm?

Answers

Answer:

s(2) = 17 cm

Step-by-step explanation:

We are told that the velocity function is;

v(t) = 6t² + 4t − 5 cm/sec

Integral of velocity gives distance.

Thus;

s(t) = ∫v(t) = ∫6t² + 4t − 5

s(t) = 2t³ + 2t² - 5t + c

We are told that s(0)=3 cm

Thus;

s(0) = 2(0)³ + 2(0)² - 5(0) + c = 3

Thus; c = 3

Thus;

s(t) = 2t³ + 2t² - 5t + 3

At t = 2 secs

s(2) = 2(2)³ + 2(2)² - 5(2) + 3

s(2) = 17 cm

Find the center and foci of the ellipse: 9x2 + 16y² + 126x + 96y + 441 = 0

Answers

Center : ( –7 , –3 )Focus 1: (–7 + √7 , –3 )Focus 2: ( –7 –√7 , –3 )

[tex] \frac{(x + 7)^{2} }{16} + \frac{(y + 3) ^{2} }{9} = 1 \\ \frac{(x - h)^{2} }{ {a}^{2} } + \frac{(y - k)^{2} }{ {b}^{2} } = 1[/tex]

a= 4 , b= 3 , k = – 3 , h = –7

The center: ( h , k ) —> ( –7 , –3 )

[tex]c = \sqrt{ {a}^{2} - {b}^{2} } = \sqrt{ {4}^{2} - {3}^{2} } \\ = \sqrt{16 - 9} = \sqrt{7} [/tex]C = √7

Focus 1: ( h + c , k ) —> ( –7 + √ 7 , –3 )

Focus 2: ( h – c , k ) —> ( –7 –√7 , –3 )

I hope I helped you^_^

Nghiệm của bất phương trình | 2x - 3| - 1 <0

Answers

Answer:

x = 1 và 2       x= 1 and 2

Step-by-step explanation:

Đầu tiên trừ đi 1 để được 2x-3 nhỏ hơn -1 sau đó bạn lập phương trình bằng 2x-3 = -1 và 2x-3 = 1 để nhận được kết quả cuối cùng là 1 và 2 do đó x = 1 và 2

First subtract 1 to get 2x-3 is less then -1 then you let the equation equal 2x-3=-1 and 2x-3=1 to get a final answer of 1 and 2 therefore x=1 and 2

In an experiment, a student is to flip a quarter 10 times and record the number of times heads appears. A group of students performs the experiment 21 times, with these results.

6 4 5 6 5 6 4 6 2 4 3 4 5 7 5 8 7 5 3 5 5

Construct a dotplot with these data and then identify the dotplot you created.

Answers

Answer:

The average, mode, and median of the results are 5, meaning that half of the time the quarter will land on heads.

Step-by-step explanation:

The required dot plot shows the result of the experiment performed by flipping a quarter 21 times.


In an experiment, a student is to flip a quarter 10 times and record the number of times heads appears. A group of students performs the experiment 21 times, with these results. 6 4 5 6 5 6 4 6 2 4 3 4 5 7 5 8 7 5 3 5 5.


What is arithmetic?

In mathematics, it deals with numbers of operations according to the statements. There are four major arithmetic operators, addition, subtraction, multiplication, and division,

What is Statistic?

Statistics is the study of mathematics that deals with relations between comprehensive data.

Here,
The dot plot has been made, for the number of outcomes and their frequencies. The dot plot gives the info about the mean mode and median.

Thus, the required a dot plot showing the result of the experiment performed by flipping a quarter 21 times.

Learn more about arithmetic here:

brainly.com/question/14753192

#SPJ2

ntroduction to Functions
Assignment Active
Creating a Function from a Mapping
The mapping shows a relationship between input and
output values.
Input
Output
Which ordered pair could be removed to make this
relation a function?
O (-5,0)
0 (-1, -3)
O (4, -2)
O (6,-1)

Answers

Answer:

To be a function, each input must only have one output. In this case, input 4 has two outputs, so (4 , -2) can be removed for it to be a function.

Let me know if you have any other questions!

The ordered pair (4,2) could be removed to make the given relation a function.

What is the relation?

A relation is a function if for every input (x-value) there is exactly one output (y-value).

If there are two or more ordered pairs with the same x-value but different y-values, then the relation is not a function. In this case, one of those ordered pairs would need to be removed in order to make it a function.

As per the question, the relation was given as {(-5,0), (2, -3), (-1, -3), (4, -2), (4, -2), (6,-1)}, the ordered pair (4,-2) would need to be removed because there are two outputs (-2 and 2) for the input of 4.

Thus, removing (4,2) would result in the function.

Learn more about the relations here:

https://brainly.com/question/29207494

#SPJ7

Read is solving the quadratic equation 0 equals X over two minus 2X -3 using the quadratic formula which shows the correct substitution of the values ABC into the quadratic formula quadratic formula X equals negative B+

Answers

Answer:

[tex]x = \frac{-(-2) \± \sqrt{(-2)^2 - 4*1*-3}}{2*1}[/tex]

Step-by-step explanation:

Given

[tex]0 = x^2 - 2x -3[/tex]

Required

The correct quadratic formula for the above

A quadratic equation is represented as:

[tex]ax^2 + bx + c = 0[/tex]

And the formula is:

[tex]x = \frac{-b \± \sqrt{b^2 - 4ac}}{2a}[/tex]

So, we have:

[tex]0 = x^2 - 2x -3[/tex]

Rewrite as:

[tex]x^2 - 2x - 3 = 0[/tex]

By comparison:

[tex]a= 1; b = -2; c = -3[/tex]

So, we have:

[tex]x = \frac{-b \± \sqrt{b^2 - 4ac}}{2a}[/tex]

[tex]x = \frac{-(-2) \± \sqrt{(-2)^2 - 4*1*-3}}{2*1}[/tex]

Find the nth term of each of the sequences.
(a) 16, 19, 22, 25, 28, ...
(b) 1,3,9,27,81,...

Answers

Answer:

a) 16, 19, 22, 25, 28, 31, 34, 37, 40

b) 1, 3, 9, 27, 81, 243, 729, 2187

Explanation:

a) Add 3 on every number.

b) Multiply every number by 3.

can someone help me with this?

Answers

Answer:

The answer is 660

Step-by-step explanation:

30×2200÷100= 660

Answer:

660

Step-by-step explanation:

to know 30%

2200÷10×3=660

What is the range of the given data set? OA) 30 OB) 32 OC) 37 OD) 40​

Answers

Answer:

Range = maximum number - minimum number

maximum number = 99minimum number = 62

Range = 99 - 62 = 37

Answer:

37

Step-by-step explanation:

The range is the difference between the highest number and the smallest number.

Looking at the stem and leaf plot we can identify the largest and smallest number

Largest number: 99

Smallest number: 62

If range = largest number - smallest number then range = 99 - 62 = 37

12. PLEASE HELP ME
Which of the following are the coordinates of the vertex of y= x2 - 10x + 2?

A. (–10, 2)

B. (2, –10)

C. (–5, 23)

D. (5, –23)

Answers

Answer:

I think b no. is the correct answer

Answer:

D. (5, –23)

Step-by-step explanation:

The vertex is in essence the turning point of the parabola y = x² − 10x + 2

the x coordinate of the turning point =  

                                                        =  

                                                        =  5

when x = 5, y = (5)² - 10(5) + 2

                     = -23

Thus coordinate or vertex is ( 5, -23)

5 right 23 down

TRIGONOMETRY
Could someone please help me with 5.2 please...it would really help alot:)​

Answers

sin(x+y) - sin(x-y) - 1 = cos(2x)

sin(90) - sin(x-y) - 1 = cos(2x)

1 - sin(x-(90-x)) - 1 = cos(2x)

-sin(2x-90) = cos(2x)

-1*(sin(2x)cos(90) - cos(2x)sin(90)) = cos(2x)

-1*(sin(2x)*0 - cos(2x)*1) = cos(2x)

-1*(0 - cos(2x)) = cos(2x)

-1*(-cos(2x)) = cos(2x)

cos(2x) = cos(2x)

This confirms the identity is true.

Notice that throughout this proof, I only changed the left hand side.

On the 5th line, I used the identity sin(A-B) = sin(A)cos(B)-cos(A)sin(B).

In right triangle ABC, AB = 3 and AC = 9. What is the measure of angle B to the nearest degree?

Answers

Answer:

90 degrees

Step-by-step explanation:

see image

make x A

y B

z C

AB=3 (given)

AC=9 (given)

measure of angel B or y, is 90

if

x= A

y= C

z= B

then the hypotenuse would be shorter than one of the legs

3<9

so B has to be the right angle (90 degrees)

Find the equation of a line perpendicular to 8x - 2y = 4 and passes through the point (4, 3).

Answers

y = -2x-3

Lines that are perpendicular have slopes that are negative reciprocals of each other. Meaning, if a line has slope  , then a line perpendicular to this has slope . That means the slope of our perpendicular line is

The equation of the line that is perpendicular to 8x - 2y = 4 and passes through the point (4, 3) is y = (-1/4)x + 4.

To find the equation of a line that is perpendicular to the line 8x - 2y = 4 and passes through the point (4, 3), we can use the fact that the slopes of perpendicular lines are negative reciprocals of each other.

First, let's rewrite the given equation in slope-intercept form (y = mx + b), where m represents the slope:

8x - 2y = 4

-2y = -8x + 4

Divide both sides by -2:

y = 4x - 2

The slope of the given line is 4.

The slope of a line perpendicular to this line would be the negative reciprocal of 4, which is -1/4.

Now, we have the slope (-1/4) and a point (4, 3). We can use the point-slope form of a linear equation:

y - y₁ = m(x - x₁)

Substituting the values, we have:

y - 3 = (-1/4)(x - 4)

Expanding and simplifying, we get:

y - 3 = (-1/4)x + 1

Adding 3 to both sides, we have:

y = (-1/4)x + 4

To learn more about equation click on,

https://brainly.com/question/20712656

#SPJ2

x = 3

x = 5

x = 0

x = 2

Answers

Answer:

x=2 is incorrect

Step-by-step explanation:

Y(2)=(3/4)*x^2=(3/4)*4=3

Which of the following is a parent function?
O A. f(x) = e*
O B. f(x) = 2.34
O x
C. f(x) = x4 +3
O D. f(x) = 2e2x

Answers

Answer:

[tex]f(x) = e^x[/tex]

Step-by-step explanation:

Given

Options (a) to (d)

Required

Which is a parent function

A parent function is such that has a single term without coefficients

From the list of given options

[tex]f(x) = e^x[/tex] suits the above definition

Other options (b) to (d) either have coefficients, or have multiple terms

find the ratio of Rs 20 and 900​

Answers

Answer:

1 : 45

Step-by-step explanation:

Given

Rs 20 and 900 ( divide both by 20 )

= 1 : 45

(x-4)°=1
giải hộ em với ạ

Answers

Answer: 5

Step-by-step explanation:

⇒ (x - 4) = 1

⇒ x = 1 + 4

⇒ x = 5

Therefore value of x = 5

Answered by Gauthmath must click thanks and mark brainliest

Geometry, please answer question ASAP

Answers

Answer:

Triangle ACB =~ triangle DFE, by adding 6 units to each side of both triangles their relationship will not change. They are still similar.

Step-by-step explanation:

The answer isn't great in all honesty but it's been a long time since I took geometry and I don't 100% remember the proper way of stating it. Though I am 100% sure they stay similar.

Sorry couldn't be of more help but figured something was better then nothing

If C.P. = Rs. 480, S.P. =Rs.528 find profit and profit percent​

Answers

In this question first you should find profit amount by using formula and you should use profit amount in profit percentage formula then you should calculate it

What is the area of the following circle?

Answers

Answer:

[tex]16\pi\:\mathrm{units^2}\text{ or }50.24\:\mathrm{ units^2}[/tex]

Step-by-step explanation:

The area of a circle with radius [tex]r[/tex] is given as [tex]A=r^2\pi[/tex].

In the question, we're given [tex]r=4[/tex] and asked to solve for [tex]A[/tex].

Substituting [tex]r=4[/tex] into [tex]A=r^2\pi[/tex], we get:

[tex]A=4^2\pi,\\A=\boxed{16\pi\:\mathrm{units^2}}[/tex]

If we use [tex]\pi=3.14[/tex] as mentioned in the question, we would have:

[tex]A=16\pi=16\cdot 3.14=\boxed{50.24\:\mathrm{units^2}}[/tex]

What is the slope of the line that contains the points (9,-4) and (1,-5)? O A. 8 O B. - O c. 1 1 O D. 8​

Answers

Answer:

1/8

Step-by-step explanation:

-5-(-4)/1-9

A wholesaler purchased an electric item for Rs 2,700 and sold to retailer at
10% profit. The retailer sold it at 20% profit to a consumer. How much did the
consumer pay for it.


PLZ PLZ HELP.......​

Answers

Step-by-step explanation:

10 % of 2700 = 270

so he had 2970 rupee

20% of 2970 = 594

so customer have to pay 2970 + 594 = 3564

Other Questions
One of Hoover's relief measures for the poor was creation of the:Works Progress administrationFarm Credit ReliefReconstruction Finance CorporationEmergency banking Act Our system is a block attached to a horizontal spring on a frictionless table. The spring has a spring constant of 4.0 N/m and a rest length of 1.0 m, and the block has a mass of 0.25 kg. Compute the PE when the spring is compressed by 0.50 m. I need help on this 20 points The required minimum cooking temperature for ground beef is 155F (68C). Why must ground beef be cooked to this temperature? This temperature kills germs that may be in the meat This temperature reduces the amount of fat in the meat People don't like to see pink inside the meat This temperature activates important nutrients in the meat PLS HELP ME ON THIS QUESTION I WILL MARK YOU AS BRAINLIEST IF YOU KNOW THE ANSWER PLS GIVE ME A STEP BY STEP EXPLANATION!!A charity is holding a benefit at an Italian restaurant. The owner of the restaurant offers to donate $200 in addition to $4 for each person that attends. Write an equation where d is the total donation and p in the number of people that attend.A. d=4p+200B. d=4+200pC. p=4+200dD. p=4d+200 What is Halo form reaction? help don't work this is easyokay tomorrow I have a assignment and is is based on which one better country side or urban (city) and I am team urban (city) so pls share your idea pls it's mean a lot to me I will mark brainist In the 1800s, mass production led to fill in the blanks with the correct form of the verbs given in the brackets ................. 1. one of the boys _________ (be) a good student 2. The policemen _______(has) solved the case Why was it significant that Japanese bombers failed to destroy several aircraft carriers in Pearl Harbor?A. It demonstrated that the Japanese did not have the power to wage a full-out war.B. The successful defense of these carriers proved the United States had the power to defeat Japan.C. The United States could use these carriers as part of its Lend-Lease program with Britain.D. The three naval carriers would play an important role in other battles throughout the war. Which of the following is a monomial? Three research departments have 6, 9,and 7 members, respectively. Eachdepartment is to select a delegateand an alternate to represent the department at a conference. In howmany ways can this be done? Please help with this, my head hurts so much and I'm sick but this is the hw due tomorrow T^T Waterway Industries was organized on January 1, 2021. During its first year, the corporation issued 2,400 shares of $50 par value preferred stock and 150,000 shares of $10 par value common stock. At December 31, the company declared the following cash dividends: 2021, $5,800; 2022, $13,100; and 2023, $28,800.Required:Show the allocation of dividends to each class of stock, assuming the preferred stock dividend is 5% and noncumulative. How can you keep yourself away from such social problems ?Mention any four aways answer. Correct answer gets brainliest and 5 stars the volume of a cube that is 6 units on an edge in exponential form Who said the following, and what does it mean? Conceit, more rich in matter than in words, Brags of his substance, not of ornament. They are but beggars that can count their worth; But my true love is grown to such excess I cannot sum up half my sum of wealth. (II. vi. 33-37) Juliet; she is saying that she doesn't care about money. Lady Capulet; she is reminding Juliet how lucky she is to be marrying Paris. Romeo; he is saying that true understanding has made him realize how very lucky he is to be marrying Juliet. Juliet; she is saying that true understanding is enriched by reality and worth more than outward appearances. sort the following numbers from least to greatest.18.5635, 18 13/16, 18.3125 PLEASE HELP MEAn expression is shown below:6x^2y 3xy 24xy^2 + 12y^2Part A: Rewrite the expression by factoring out the greatest common factor. (4 points)Part B: Factor the entire expression completely. Show the steps of your work. (6 points)